Exercice proba

Aide sur les questions de probabilités.
harith
Membre
Messages : 10
Inscription : 22 avril 2017, 21:45

Exercice proba

Message par harith » 26 avril 2017, 20:17

Il ya 2 exercice vous pouvez m'aidez s'il vous plait.
Pièces jointes
IMG_3080 (1).JPG
IMG_3080 (1).JPG (99.23 Kio) Consulté 3151 fois

Avatar de l’utilisateur
Job
Propriétaire du forum
Messages : 2584
Inscription : 28 juin 2013, 15:07
Contact :

Re: Exercice proba

Message par Job » 27 avril 2017, 15:58

Bonjour

Exercice 4
Soit $T$ la variable aléatoire "taux de cholestérol" et $X$ la variable aléatoire suivant la loi normale centrée réduite.
$X=\frac{T-m}{\sigma}$

$P(T<165)=0,56$ et $0,56=P(X<0,15)$ donc $\frac{165-m}{\sigma}=0,15$
$P(T<180)=0,90$ et $0,90=P(X<1,28)$ donc $\frac{180-m}{\sigma}=1,28$

On résout le système formé par les 2 équations, ce qui donne $m=163$ et $\sigma = 13,27$

On trouve alors que $P(T<182)=0,9236$ donc $P(T>182)=0,0764$ et donc, sur 10000 personnes, il faut prévoir de soigner 764 personnes.

Avatar de l’utilisateur
Job
Propriétaire du forum
Messages : 2584
Inscription : 28 juin 2013, 15:07
Contact :

Re: Exercice proba

Message par Job » 27 avril 2017, 16:23

Exercice 3

Je n'ai pas vraiment de compétences en Statistiques donc mes réponses sont à prendre avec beaucoup de réserves.

1. (a) $E(X_i)=1\times p +0\times (1-p)=p$
$E(X_i^2) =1\times P +0\times (1-p)=p$ donc $V(X_i)=E(X_i^2)-(E(X_i))^2=p-p^2$

(b) $F_n$ = moyenne de l'échantillon ou moyenne empirique (voir votre cours)
L'espérance est linéaire donc $E(F_n)=\frac{1}{n} \times (np)=p$
$V(F_n)=\frac{1}{n^2}\times n(p-p^2)=\frac{p-p^2}{n}$

(c) $F_n$ est un estimateur sans biais (voir votre cours)

(d) On peut approcher la loi de $F_n$ par la loi normale de moyenne $p$ et d'écart-type $\frac{\sqrt{p(1-p)}}{\sqrt n}$

2. $I=\left[p-1,96\frac{\sqrt{p(1-p)}}{\sqrt n}, p+1,96\frac{\sqrt{p(1-p)}}{\sqrt n}\right]$
Avec $p=0,47$ et on remplace $n$ par chacune des valeurs donnée.

4. Intervalle de fluctuation : $\left[p-u_{\alpha}\frac{\sqrt{p(1-p)}}{\sqrt n} , p+u_{\alpha}\frac{\sqrt{p(1-p)}}{\sqrt n}\right]$ avec $p=0,52$ et $P(-u_{\alpha}\leq X \leq u_{\alpha})=\alpha$ ($X$ suivant la loi normale centrée réduite).

Avec $\alpha =0,98$, on a $u_{\alpha}=2,325$ et on remplace par les différentes valeurs.

Répondre